Тёмный
Geendle
Geendle
Geendle
Подписаться
Geenddle is a mathematics channel with the aim of bringing knowledge to you, covering all possible subjects and helping people from all over the world to have access to the study of this essential tool for the development of science and for our daily lives. It is a channel that aims to serve students, teachers, enthusiasts and lovers of mathematics, and at the same time find fun while discussing our challenges.

We hope you feel very comfortable on our channel, expressing your comments and leaving your suggestions as well.

We invite you to subscribe to our channel and enjoy this incredible journey through never-ending science and truly "Go beyond math"

We appreciate you!
Geendle
Functional Equation | Geendle
17:45
21 день назад
Math olympiad equation | Geendle
8:11
Месяц назад
Math Olympiad Great Challenge | Geendle
11:53
Месяц назад
Комментарии
@user-dq3uh6ee5w
@user-dq3uh6ee5w 15 дней назад
3.
@slavinojunepri7648
@slavinojunepri7648 21 день назад
Excellent approach. I like the way you factor the left hand side of the equation. You could have considered all factorizations of 1241 to show that 3 is the only solution. You used the factorization 17x73 to show that 3 is a solution of the equation.
@MrKA1961
@MrKA1961 27 дней назад
You sort of "guessed" the solution. Why not a direct guess: for m=2 the LHS is far less than 559, then try m=3 and you're done.
@qnicks23434
@qnicks23434 27 дней назад
What happens when x equal to 4, or 12, or 4/3? When a equals to 1?
@ArtemKreimer
@ArtemKreimer 27 дней назад
The solution as presented needs to exclude the points at x=4, x=12, and x=4/3. But it also needs to separately include the cases for x=-2 and x=2, which are missing as is.
@ArtemKreimer
@ArtemKreimer 27 дней назад
I guess a=-2 and a=2 happen at exactly points where x needs to be excluded anyway, so my second clause doesn't really matter.
@archangecamilien1879
@archangecamilien1879 29 дней назад
x=2 works...
@DonRedmond-jk6hj
@DonRedmond-jk6hj Месяц назад
How do you know that 7^(m/3) + 6^(m/3) is an integer? If it isn't, then your solution fails.
@MARTINWERDER
@MARTINWERDER 19 дней назад
7^1 + 6^1 = 13, and 559 = 13 x 43
@jacopomaccione7791
@jacopomaccione7791 Месяц назад
Hi, I also solved it with the same method but how do you prove that 823543=7^7? In my solution I use this idea: the last term is 3 so the repeated number could be 3 or 7 because they are the only numbers that if repeated generate a 3: for 3 the pattern is 3^(1+4k) and for 7 it is 7^(3+4k). So 3^3 is not a solution because 1+4k cannot be 3 (if k is natural), but if you do it for 7 you find the solution because 3+4k is equal to 7 when k is 1, so the result is 7 ^7. The idea could also be extended to numbers that contain 3 or 7 as a final term such as 13, 27 or 10023...I thought that with 10^10 we exceed the digits of 823543 and this allows us to stay under 10
@theupson
@theupson Месяц назад
because f(a,b) = f(b,a), i started by using a = 2+ x, b=2 -x ... solutions for x will be in equal and opposite pairs, and this substitution into the second given will result in a quartic. that quartic must have factors of the form (c - x^2) only because of the first observation, meaning it is really quadratic in x^2. distributing and combining (2+x)^5 + (2-x)^5 is demeaning and time-consuming, but is most of the work in the problem.
@walterwen2975
@walterwen2975 Месяц назад
Math Olympiad Problem: 7^m + 6^m = 559; m = ? 7^m + 6^m = 559 = (13)(43) = (7 + 6)(49 - 6) = (7 + 6)(7² - 6) = 7³ + 6(7²) - 6(7 + 6) = 7³ + 6(7² - 13) = 7³ + 6(36) = 7³ + 6(6²)= 7³ + 6³; m = 3 Answer check: 7^m + 6^m = 559; Confirmed as shown Final answer: m = 3
@Geendle
@Geendle Месяц назад
www.youtube.com/@walterwen2975, Wow! That's a nice solution!!! I'm saving it and I will post a video solving a similar problem in two different ways soon. Thank you!
@walterwen2975
@walterwen2975 Месяц назад
@@Geendle Thanks, 🙏
@HoSza1
@HoSza1 Месяц назад
@@walterwen2975 ok you found a solution after quite a bit of mental juggling and how do you know you got all the solutions? That part is still missing tho.
@walterwen2975
@walterwen2975 Месяц назад
@@HoSza1 Be glad to know if there is any more solution.
@DOTvCROSS
@DOTvCROSS Месяц назад
If a person learns sequences of quartics, just like most remember sequences of squares, {1,4,9,...}. By mental recall of memorized arithmetic: Whatever the sum be, the sum of 6 and 7's as squares will be too small, as a sum of quartic's, HA, a single 6 OR 7 is too much. Edit start: If that has been said already: I paused at @0:03 - took longer to type then solve.
@Geendle
@Geendle Месяц назад
Thank you!!! This would be a nice solution too!
@foerza649
@foerza649 Месяц назад
just brute force it 😂. 559 is not really a big number
@Geendle
@Geendle Месяц назад
🤣🤣🤣🤣🤣 you're funny! I Love your comment!
@krwada
@krwada Месяц назад
prime factorization gets answer very fast 1156 = 2x2x17x17 =34^2
@Geendle
@Geendle Месяц назад
Yes!!!!
@agentm10
@agentm10 Месяц назад
This is probably the easiest olymp8ad problem ever.
@manudude02
@manudude02 Месяц назад
They should do one of these where it isn't conveniently an integer.
@Geendle
@Geendle Месяц назад
I'll solve one of this soon! thanks!
@HoSza1
@HoSza1 Месяц назад
6, 36, 216. 7, 49, 243. m=3. Is there anorher solution? No, because f(x)=6^x+7^x is a function that increases monotonically. Yes calculus. 😂
@Geendle
@Geendle Месяц назад
😍 Thank you!
@keescanalfp5143
@keescanalfp5143 15 дней назад
yeah who was, at secondary school, advised to memorize • all squares from 11 up to 20 . • all powers under 1024 of 2 up to 10 . so incl. 2¹⁰ , 3⁶ , 4⁵ , 5⁴ , 6³ ,7³ , 8³ , 9³ . that is , in the question is immediately visible 343+216. then indeed considering that 6^m and 7^m are monotonously increasing functions of m, meaning that the found m = 3 should be the only possible answer .
@tortinwall
@tortinwall Месяц назад
Easy. 559 is odd so m must also be odd as 6^m is even. 7^5 is far too big so m must be 3. I’ll save time and not watch the video.
@keescanalfp5143
@keescanalfp5143 15 дней назад
well that's funny . and wouldn't 7^4 be odd . could you please show whether it will be even . . btw it is equal to 2401, like 49² .
@lucho2868
@lucho2868 Месяц назад
32^2=1024 and 33^2 is odd so (asuming the answer is an integer number) 34^2=1156
@kicorse
@kicorse Месяц назад
I had the same reasoning but verified it by noting that 1024 + 4*33= 1156. In general, x^2+4(x+1) = (x+2)^2.
@lucho2868
@lucho2868 Месяц назад
To verify you can just multiply with the standard algorythm: 34^2 = 30*34+4*34 = 1020+136 = 1156 . There's no need of looking for algebraic identities since it's an arithmetic problem that doesn't require further generalisation.
@kicorse
@kicorse Месяц назад
@@lucho2868 of course you can, but if you know rules for nearby perfect squares than my way is quicker. You might as well say that you didn't need to use your knowledge that 32^2 = 1024 in the first place!
@lucho2868
@lucho2868 Месяц назад
@@kicorse You also used 1024^.5=32. In fact you wrote a proof around the fact that that 32^2=1024. And your answer is formally incorrect since you didn't even proove that 32^2=1024. Your solution is useless because it requires differences, elementary algebra and (over all that) a long mutiplication whilst mine only require the long multiplication to check at the end. Making it shorter, more rigurous, and more generally applycable than yours.
@lucho2868
@lucho2868 Месяц назад
@@kicorse for example, if you wanted approximate the value of any square root by hand the long multiplication's result of a nearby approximation could be used as an input in a Bakhshali-kind formula to get a good approximation result whilst the method you suggest would work less generally.
@lightning77125
@lightning77125 Месяц назад
34
@Geendle
@Geendle Месяц назад
That's rigth!
@beyondoftrue
@beyondoftrue Месяц назад
Nice.
@Geendle
@Geendle Месяц назад
Thanks, bro!
@ben_adel3437
@ben_adel3437 Месяц назад
1156 is between 900 and 1600 and squared numbers that end in 6 are either X4 or X6 so the answer is either 34 or 36 and after that you just need to check these
@Geendle
@Geendle Месяц назад
Thank you! That's a nice way too!
@willvu3581
@willvu3581 Месяц назад
You have to state that a, b, and c are natural numbers as well
@Geendle
@Geendle Месяц назад
Thanh you bro! I'll consider that in the next videos.
@jacopomaccione7791
@jacopomaccione7791 Месяц назад
20=(8343-7^a)^(1/a), now i think this, if 20^k (k is a natural number) finishes with a 0 for all k, it means that 8343-7^m has to finished with a 0. So 7^m has to finished with a 3 because the last 3 of 8343 minus the last term of 7^m has to be equal 0. Now I study the pattern of 7^k and it is: 7^1=7, 7^2=...9, 7^3=...3, 7^4=...1, 7^5=...7. Now i know that every 4 times the pattern restart and i know that for 7^3 i have a number that finished with a 3. So 7^a=7^(3+4n) where n is natural. But for n>1 8343-7^a is negative so my only option is n=0 that means a=3 and if you try to put a=3 in 20^a+7^a it becomes 8343
@Geendle
@Geendle Месяц назад
Well done! Thank you! Your method your method is awsome.
@jacopomaccione7791
@jacopomaccione7791 Месяц назад
@@Geendle Thank youuuuuuu
@atanasiumirela7187
@atanasiumirela7187 Месяц назад
i did it in my head in like 20 seconds when I saw the thumbnail. It's really fun!
@Geendle
@Geendle Месяц назад
That's nice Thank you!
@jacopomaccione7791
@jacopomaccione7791 Месяц назад
Niiiiiiceeeee solution, i find another process: 6=(559-7^m)^(1/m), now i think this, if 6^k (k is natural number) 6^k has to finished with a 6, for example 6*6=36, 36*6=216,..., it means that 559-7^m has to finished with a 6. So 7^m has to finished with a 3 because the 9 of 559 minus the last term of 7^m has to be equal 6. Now I study the pattern of 7^k and it is: 7^1=7, 7^2=...9, 7^3=...3, 7^4=...1, 7^5=...7. Now i know that every 4 times the pattern restart and i know that for 7^3 i have a number that finished with a 3. So 7^m=7^(3+4n) where n is natural. But for n>0 559-7^m is negative so my only option is n=0 that means m=3 and if you try to put m=3 in 6^m+7^m it becomes 559
@Geendle
@Geendle Месяц назад
Very good! I'l try to explain one similar video using your idea! Thank you!
@jacopomaccione7791
@jacopomaccione7791 Месяц назад
@@Geendle i would be honored
@gerryiles3925
@gerryiles3925 Месяц назад
1156 is a very long way from being a "huge number". As others have pointed out, this answer takes only a few seconds to work out in your head (divide by 2, divide by 2, left with 289 which is 17^2, answer is 2*17). Also, you didn't need to multipy by 4/4, what you had was (10^2 + 2) ^ 2, giving the result of 102/3 after the square root which is 34. Also, also, what's with the unrelated #-tags? The only one relevant to this video is #squareroot...
@Geendle
@Geendle Месяц назад
Thanks! the other hashtags are intended for the video to reach more people.
@TheEulerID
@TheEulerID Месяц назад
My approach is simply to find the prime factors. It's obviously divisible twice by 2, leaving us only to find the square root of 289. It's also obviously not divisible by 3. 5 can be rules out as 5^4 is 625 which means we must hop that there's an exact square root of 289, or it gets very messy. It's clearly greater than 13 squared (169) which most people know, and the next prime is then 17, the square of which is 289. So, we have sqrt(2*2*17*17) = 34. nb. I don't think merits being an olympiad question.
@Geendle
@Geendle Месяц назад
Nice! Thank you. I was just trying to show another way of solving it.
@Cynicalgeek743
@Cynicalgeek743 Месяц назад
Took me a whole 5 seconds to solve. 9mins and 7 secs of my life saved not following the solution
@Geendle
@Geendle Месяц назад
Nice!😉
@edmx
@edmx Месяц назад
This is mental. So slow! Anyone doing a math olympiad would just look at it and know the answer, surely?
@Geendle
@Geendle Месяц назад
Probably yes!
@mouradbelkas598
@mouradbelkas598 Месяц назад
You had 7^(m/3) + 6^(m/3) = 13, hence 13 = 6+7 then m/3 = 1 and m=3
@Geendle
@Geendle Месяц назад
Yes, that's a beautiful observation! Thank you!
@ericfielding668
@ericfielding668 Месяц назад
I used the "by hand" method, which only took two steps = less than a minute. Grade 4 students used to be taught this method, which is similar to long division.
@Geendle
@Geendle Месяц назад
Thats great! Thank you!
@Krmpfpks
@Krmpfpks Месяц назад
It is waay simpler to solve by converting it to binary. Find the largest power of 2 that fits in 400 -> 2^8 = 256 Same for the remaining 144 -> 2^7 = 128 Same for the remaining 16 -> 2^4 The approach shown in the video serves no better purpose, it’s neither faster nor more general
@beyondoftrue
@beyondoftrue Месяц назад
I also thought the same way. When I realized that the base is two, I intuitively remembered converting decimal numbers to binary.
@Geendle
@Geendle Месяц назад
I'll post another video using this method! Thank you!
@user-zr7ie6so1q
@user-zr7ie6so1q Месяц назад
a,b,c \in {4,7,8} would be better
@Geendle
@Geendle Месяц назад
Thanks! I will improve!
@_.__-._-_.-..-...
@_.__-._-_.-..-... Месяц назад
I tried beginning to solve it before opening the video, my method was to try to find the prime factors of 1156 to if possible remove most of the work from the square root, found that you could divide it by 2 two times wich gives 2√289, then I had to think for a bit longer until I reached 17 wich fits perfectly, giving √1156 = 34, ngl I was not expecting the answer to be an integer
@Geendle
@Geendle Месяц назад
Nice! Thank you!
@JohnMackenroth-mg6jc
@JohnMackenroth-mg6jc Месяц назад
I used the method I learned in freshman year of high school. I had the answer in less than a minute.
@Geendle
@Geendle Месяц назад
Wow! that's nice!
@triforce9856
@triforce9856 Месяц назад
I was way faster by testing numbers
@francoismusic_
@francoismusic_ Месяц назад
It is not.
@triforce9856
@triforce9856 Месяц назад
@@francoismusic_ But its literally just taking the max. allowed exponent 3 times
@tominmo8865
@tominmo8865 Месяц назад
What?!?! I figured out that it is 34 in about 2 seconds, in my head. 30 x 30 = 900.....40 x 40 = 1600. So the number must fall between 30 and 40. Only 4 squared and 6 squared end in 6, so it has to be one of those two numbers in the singles position. Since 1156 is much closer to 900 than 1600, the singles number must be 4. Therefore 34 is the answer. Multiplied 34 x 34 to verify--also in my head. I am not a mathematician or an engineer, and I am 74 years old. Your method is far too complicated. I stopped watching after about a minute.
@krabkrabkrab
@krabkrabkrab Месяц назад
I did too. I used 32^2=1024; well-known if you're familiar with powers of 2. So it's higher than 32, but ends in 6, so the rest is as you stated.
@Geendle
@Geendle Месяц назад
That's nice! Thank you!!!
@ericsonmario5337
@ericsonmario5337 Месяц назад
🎉
@Geendle
@Geendle Месяц назад
Thanks pidimo!😉
@ronbannon
@ronbannon Месяц назад
You msde a mistake, the real solutions are (5, 2) amd (-2,-5) You are also making a mistake with the complex solutions, x^2 -3x+19=0, x= 3/2 \pm i sqrt(67)/2.
@Geendle
@Geendle Месяц назад
Thanks! I'll check that!
@MatsCooper
@MatsCooper Месяц назад
The terms multiply to 17 therefore one of the terms is 17 and the other must be one. Such nonsense!!
@Geendle
@Geendle Месяц назад
Yes! Thank you!
@ericsonmario5337
@ericsonmario5337 Месяц назад
🎉
@AA-st6of
@AA-st6of Месяц назад
how about this: let 3^(2x-1/x+1) = t 3^(3x-x-1/x+1)=t 3^(3x/x+1)=3t thus, 3^(5x-1/x+1) = t・3t =3t^2 which means that 3t^2+t-30=0 t must be real so the only solution is t=3 hence 2x-1/x+1=1 i.e. x=2
@Geendle
@Geendle Месяц назад
www.youtube.com/@AA-st6of, Thank you! This is really nice! I'll try this method in a different video!!!
@AA-st6of
@AA-st6of Месяц назад
@@Geendle 👍
@Essentialsend
@Essentialsend Месяц назад
on the one hand I don't like problems where imagination has go soooo far. I am a teacher and would not have found every step. Would have solved the quadratic in a different way. However. This is an example I really like to teach my students. It's too complex to solve it alone. but its a perfect example to train all different kind of methods and keep an eye open to all of them at once. Learning this problem by heart is in itself a good lesson. So thank you. I copied it already in latex to make a lesson out of it.
@Geendle
@Geendle Месяц назад
www.youtube.com/@Essentialsend,Thank you!😍 I will bring some more similar problems to share ideas about them.
@uncannyroaches5933
@uncannyroaches5933 Месяц назад
27 and 3 are the only possible values. so equate powers to (3,1) and (1,3) and see which value yields a coherent solution. thjnk about it powers of 3 are 1,3,9,27. any power higher than that cannot yield a sum of 30. and we cannot have irrational numbers as the sum of two irrational numbers canot be rational(integer). so fractional powrrs are out of the picture. and, negative powers are also out of picture. you cannot possiblt have 1/3 and 89/3, both are not power of 3.
@Geendle
@Geendle Месяц назад
www.youtube.com/@AA-st6of, you are right in your analysis. Thank you!!!
@ronnietwelvetoes1876
@ronnietwelvetoes1876 Месяц назад
Borring
@ThoMas-dq7pg
@ThoMas-dq7pg Месяц назад
x=1 didnt work so x=2 was my next guess
@Geendle
@Geendle Месяц назад
That's right!!!
@pukulu
@pukulu Месяц назад
x =2 by inspection. The technical solution is preferable of course.
@Geendle
@Geendle Месяц назад
That's right. Thank you!!!
@tejpalsingh366
@tejpalsingh366 Месяц назад
On solving 4^2-4&(-3)^2-(-3) gives solns .. but power value cant be -ve hence first x=2 only works
@Geendle
@Geendle Месяц назад
Yes! That's the answer. Thanks!
@user-bx8tu3sj3w
@user-bx8tu3sj3w Месяц назад
Congratulations ! What does it means? The ''factors'' of 175 that lead us to a positive integer solution are just 7 and 25. Thanks!
@Geendle
@Geendle Месяц назад
www.youtube.com/@user-bx8tu3sj3w, that's it! Thank you!
@agranero6
@agranero6 Месяц назад
A Physicist(or any common sense person) will realize the positive part just crescent and so x should be small, just throwing integers will find x=2 and it is the only one. Even if x was not integer is just throw integers on it until the left size would be bigger than the left. Why spend 12 minutes at it? It takes about 7 to 10 seconds. You don't need to be a Ramanujan to solve this.
@user-yt4fn9pj8l
@user-yt4fn9pj8l Месяц назад
Sorry man, but when you are rewriting to (a-b)*(a+b)=12 and saying that (a-b) = 1 or 2 or 3 and (a+b) = 12 or 6 or 4 your are making mistake. (a-b) or (a+b) are integer values only for even values of 'x'. So you are solving the equation for even 'x'. If you take odd 'x' then (x^x)/2 will be non-integer, e.g. x=3 -> (a+b) = 3^13.5+3^1.5; or different value: x=5 -> (a+b) = 5^1562.5+5^2.5. This isn't integer. Yeah, you got lucky with this math problem that both your multipliers (a+b) and (a-b) are integers and that for growing x>1 you get ever growing function of f(x)=x^x^x-x^x. But for another similar problem like x^x^x-x^x=7625597484960 this your trick wouldn't work. (a+b)*(a-b)=7625597484960 doesn't imply, that (a+b) is integer and (a-b) is integer. For this case, where x=3, this will be 2761453.635828058 * 2761443.2435232126 = 7625597484960.
@Geendle
@Geendle Месяц назад
I like your analysis. Thank you for sharing your ideas!